ABFM HEART DISEASE

¡Supera tus tareas y exámenes ahora con Quizwiz!

An active 66-year-old female presents with intermittent chest pain and dyspnea. She is currently pain free. A resting EKG is normal.If found on the history and examination, which one of the following symptoms is most likely to be associated with myocardial ischemia as the cause of chest pain? An episode of diaphoresis associated with the chest pain Pain reproduced by chest wall palpation on the left side of the chest Pain that comes and goes with and without exertion Intermittent pleuritic-type pain and dyspnea

A Cardiac ischemia is classically defined as deep, poorly localized chest or arm discomfort reproducibly associated with exertion or emotional stress. It is relieved with rest and nitroglycerin. It can present in an atypical fashion, and the discomfort can localize or radiate to the neck, lower jaw, throat, shoulder, epigastrium, hands, or upper back. It may be entirely absent in some cases. In older patients without chest pain, new-onset or unexplained exertional dyspnea is the most common anginal equivalent, even with a normal resting EKG.Although they may be present, pleuritic-type pain, pain reproduced with movement or palpation of the chest wall or arm, and sharp or stabbing pain are not characteristic features of myocardial ischemia. Very brief episodes of pain, lasting a few seconds or less, are also not characteristic of myocardial ischemia. In a meta-analysis of symptoms useful in diagnosing acute coronary syndrome in a low-risk setting, diaphoresis was found to be the strongest predictor of myocardial infarction (MI) (likelihood ratio [LR] = 2.44), and the presence of chest wall tenderness significantly reduced the possibility of MI (LR = 0.23). A completely normal EKG does not exclude the possibility of acute coronary syndrome because 1%-6% of such patients eventually are found to have an acute myocardial infarction (non-ST-segment elevation by definition) and at least 4% have unstable angina.

An otherwise healthy 53-year-old male presents with episodes of substernal chest pain that occur both at rest and with exertion. His father had a myocardial infarction at age 58. A resting EKG shows a right bundle branch block. A potassium level and hemoglobin level are both normal.Which one of the following would be the most appropriate next step to evaluate this patient for coronary artery disease? Transthoracic echocardiography Coronary CT Adenosine myocardial perfusion imaging Ambulatory EKG (Holter) monitoring

A The 2019 European Society of Cardiology guidelines for the diagnosis and management of chronic coronary syndromes recommend echocardiography as the initial step to ascertain cardiac function. Echocardiography is important for excluding alternative causes of chest pain, as well as for detecting concurrent cardiac diseases such as valvular heart disease, heart failure, and most cardiomyopathies. However, it is important to remember that these diseases often coexist with obstructive coronary artery disease. Coronary CT might be indicated based on the findings from echocardiography. Adenosine perfusion testing would not be the next step in the evaluation for most patients. Holter monitoring would be indicated if there were a history of syncope, near syncope, palpitations, or tachycardia.

A 58-year-old female with heart failure has an ejection fraction of 45%. She is currently asymptomatic and her medications include lisinopril (Prinivil, Zestril), 40 mg daily; carvedilol (Coreg), 6.25 mg twice daily; and furosemide (Lasix), 40 mg daily. Her blood pressure is 128/84 mm Hg and her pulse rate is 74 beats/min. She has no edema on examination and her lungs are clear. Laboratory testing shows a slightly elevated pro-BNP level.Which one of the following would be most appropriate at this time? Increase the dosage of carvedilol to 12.5 mg twice daily Stop lisinopril and start ramipril (Altace), 10 mg daily Start losartan (Cozaar), 50 mg daily Start metolazone, 5 mg three times a week Start spironolactone (Aldactone), 25 mg twice daily

A The patient is on a very low dosage of carvedilol. Maximizing the dosage of β-blockers is crucial in heart failure treatment and should be considered even if the patient is asymptomatic and vital signs are normal. Guidelines recommend that patients who have heart failure with reduced ejection fraction should have their medical therapy titrated to target dosages.Adding metolazone is not necessary unless the heart failure has been refractory to the use of a loop diuretic. Metolazone can also cause significant potassium wasting and should be used with caution, especially in patients currently on loop diuretics.Spironolactone has a long half-life so twice-daily dosing is not recommended because it may increase hyperkalemia without adding any benefit. Dosages greater than 25 mg daily are not recommended for heart failure.Losartan should not be added because trials have shown worse outcomes when an angiotensin receptor blocker (ARB) is added to an ACE inhibitor. Patients who cannot tolerate an ACE inhibitor should be switched to an ARB but this patient is tolerating the ACE inhibitor. It is more useful to add or maximize β-blocker therapy with either an ACE inhibitor or an ARB, rather than to combine all three types of agents. This patient's lisinopril dosage is already at the target level and she is tolerating it so there is no reason to switch to another ACE inhibitor.

A 62-year-old female is admitted to the hospital with an ST-elevation myocardial infarction treated with fibrinolytic therapy because you are unable to transport her to a cardiac catheterization lab. She develops the arrhythmia shown on the tracing below, but is asymptomatic and has stable vital signs.Which one of the following interventions would be most appropriate? Observation only Amiodarone Lidocaine (Xylocaine) A magnesium infusion A temporary pacemaker

A This is a tracing of an accelerated idioventricular rhythm (AIVR), which is characterized by a wide QRS complex with a regular rate that is higher than the atrial rate and lower than 100 beats/min. Its appearance is an inexact indicator of reperfusion. Treatment is not indicated and suppression of the rhythm may lead to hemodynamic compromise.While AIVR has been shown to have low sensitivity (45%) and specificity (64%) as a predictor of successful thrombolysis, early AIVR is a reliable sign of successful thrombolysis (P <0.05). The sensitivity of early AIVR is low (45%); however, the specificity (94%) and positive predictive value (94%) are very good. Early AIVR can therefore be used as an additive criterion to ST-segment resolution as a noninvasive marker of successful thrombolysis with streptokinase.

A 74-year-old male sees you for routine follow-up for heart failure. He is currently taking lisinopril (Prinivil, Zestril), 20 mg daily; metoprolol succinate (Toprol-XL), 100 mg daily; and apixaban (Eliquis), 5 mg twice daily. His blood pressure is 136/86 mm Hg. Based on the results of today's evaluation, you increase his dosage of lisinopril to 30 mg daily.Which one of the following sets of serum levels should be obtained in the next 1-2 weeks to assess the safety of this dosage change? Magnesium and creatinine Potassium and creatinine Sodium and creatinine Sodium and magnesium Sodium and potassium

B ACE inhibitors diminish renal reabsorption of sodium by decreasing levels of aldosterone. Because sodium is reabsorbed in exchange for potassium, renal potassium loss declines. Additionally, glomerular filtration is affected by angiotensin-mediated efferent renal arteriolar vasoconstriction. ACE inhibitor-mediated declines in angiotensin can therefore worsen renal function. When the dosage of an ACE inhibitor is changed it is therefore important to assess serum potassium and creatinine.

In patients with suspected coronary heart disease, graded treadmill exercise testing has been shown to be unreliable or inappropriate in those who are taking metoprolol succinate (Toprol-XL), 100 mg daily have left bundle branch block have bifascicular block with a right bundle branch block and left anterior hemiblock have aortic stenosis with a valve area of 1.6 cm2 had an uncomplicated acute myocardial infarction 5 days earlier

B Absolute contraindications to exercise testing include an acute myocardial infarction within the previous 2 days, unstable angina, cardiac arrhythmias causing symptoms or hemodynamic compromise, severe aortic stenosis, symptomatic decompensated heart failure, acute pulmonary embolism or infarct, acute pericarditis or myocarditis, acute aortic dissection, and lack of advanced cardiac life support equipment or signed consent. Severe aortic stenosis typically is associated with an aortic valve area (AVA) <1.0 cm2. An AVA of 1.5-2.0 cm2 is generally classified as mild aortic valve stenosis, and an AVA of 1.0-1.5 cm2 as moderate aortic stenosis. Exercise stress electrocardiography (ESE) is unreliable as a test for obstructive coronary artery disease (CAD) if the patient has left bundle branch block (LBBB).Current guidelines support ESE for evaluation of suspected obstructive CAD in ambulatory patients with LBBB. However, the specificity of ESE for reliably excluding significant occlusive coronary artery disease is significantly reduced. ESE appears to be a suboptimal test for the evaluation of obstructive CAD in patients with resting LBBB. About 50% of these patients will have an abnormal response, but in the majority of these cases there is a globally abnormal contraction that is not related to obstructive CAD. Alternative testing should therefore be considered for investigating suspected obstructive CAD in patients with resting LBBB.Patients who are taking β-blockers can safely undergo cardiac stress testing on a treadmill, although it should be noted that these patients may not be able to attain 85% of their age-related heart rate with exercise. A 2016 study showed that patients taking β-blockers who achieved 65% of their age-predicted maximal heart rate had a similar adjusted mortality rate compared to those not taking β-blockers who achieved 85% of their age-predicted maximal heart rate (P >0.05). Bifascicular block (right bundle and left anterior hemiblock) does not preclude cardiac stress testing.

You see a 63-year-old female for follow-up 2 months after coronary artery bypass graft (CABG) surgery. In addition to clopidogrel or a similar antiplatelet medication, which one of the following should you recommend to reduce the repeat revascularization rate following CABG surgery? Aspirin and β-blockers Aspirin and statin therapy β-Blockers and statin therapy Postmenopausal hormone therapy and statin therapy

B Aspirin has been shown to significantly reduce vein graft closures through the first postoperative year. According to current guidelines it should be continued indefinitely, given its benefit in preventing subsequent clinical events. After off-pump coronary artery bypass graft (CABG) surgery, dual antiplatelet therapy should be administered for 1 year using a combination of aspirin, 81-162 mg daily, and clopidogrel, 75 mg daily, to reduce graft occlusion. Aggressive statin therapy following CABG has been shown to result in less disease progression in saphenous vein grafts and to reduce the repeat revascularization rate. The American Heart Association recommends high-intensity statin therapy (atorvastatin, 40-80 mg daily, or rosuvastatin, 20-40 mg daily) after surgery for all CABG patients <75 years of age and moderate-intensity statin therapy for patients intolerant of high-intensity statin therapy and those >75 years of age. Hormone therapy and β-blockers have not been shown to affect the revascularization rate. Postmenopausal hormone therapy (estrogen/progesterone) should not be given to women undergoing CABG (SOR B).

A 70-year-old female presents with dyspnea on exertion but no chest pain or tightness. Which one of the following would be associated with findings consistent with heart failure with reduced ejection fraction even if the patient has a normal ejection fraction on echocardiography? Ischemic heart disease Mitral valve regurgitation Type 2 diabetes Hypothyroidism

B Depending on the diagnostic criteria used, it has been estimated that 20%-60% of patients with heart failure have a relatively normal left ventricular ejection fraction, arising from reduced ventricular compliance and abnormal diastolic function. Patients with mitral valve disease may have a normal ejection fraction but limited cardiac output, given the regurgitation into the atria. Estimation of true cardiac output can be greatly affected by mitral valve disease. In a patient with heart failure found to have a normal left ventricular ejection fraction, other diagnostic possibilities should be excluded, such as primary valvular disease. Heart failure can arise from high metabolic demand (high output states), and anemia, hyperthyroidism, and arteriovenous fistulae should be considered as possible causes. Other considerations in the differential diagnosis include restrictive heart disease such as amyloidosis, sarcoidosis, or hemochromatosis; pericardial constriction; episodic left ventricular systolic dysfunction; severe hypertension; chronic pulmonary disease with right heart failure; atrial myxoma; and obesity.

A 62-year-old male sees you for follow-up 1 month after an ST-elevation myocardial infarction (STEMI) and placement of a stent. He says he is having a difficult time concentrating and he feels unhappy much of the time. You suspect depression.Which one of the following is true regarding depression following a myocardial infarction (MI)? It occurs in approximately 10% of patients within a year after an MI Treatment of depression after STEMI improves mortality Cardiac rehabilitation does not change the outcomes of patients with depression SSRIs should be used with caution because of their quinidine-like effect on cardiac conduction

B Depression is a common consequence of ST-elevation myocardial infarction (MI), with major depression occurring in 15%-20% of patients. In a study lasting 29 months post MI, the risk of death or recurrent MI was significantly lower in patients taking SSRIs. Depression is also associated with poor compliance with risk-reducing treatment recommendations, abnormalities in autonomic tone that may increase susceptibility to ventricular arrhythmias, and increased platelet activation.Cardiac rehabilitation programs, social support, cognitive-behavioral therapy, and SSRIs are useful in the management of depression occurring in the year following an MI. Tricyclic antidepressants are not favored because of concerns related to their tendency to increase resting heart rate, produce orthostatic hypotension, and alter intracardiac conduction and susceptibility to ventricular arrhythmias. They have a quinidine-like effect on cardiac rate and conduction, and they should be avoided in patients with ventricular arrhythmias and ischemic heart disease.

A 61-year-old male sees you for a routine annual evaluation. A review of systems is notable only for nocturia 1-2 times per night. He has a history of a non-ST-elevation myocardial infarction 2 years ago treated with a drug-eluting stent. His current medications are metoprolol tartrate (Lopressor), 50 mg twice daily; hydrochlorothiazide, 25 mg daily; atorvastatin (Lipitor), 40 mg daily; aspirin, 81 mg daily; and docusate as needed. He is a nonsmoker. His blood pressure is 132/82 mm Hg. A physical examination is normal.Which one of the following medications is indicated at this time? Diltiazem (Cardizem) Enalapril (Vasotec) Furosemide (Lasix) Losartan (Cozaar) Spironolactone (Aldactone)

B Despite the absence of symptoms and a left ventricular ejection fraction within the normal range, this patient's previous myocardial infarction (MI) is evidence of structural heart disease, making his American College of Cardiology/American Heart Association (ACC/AHA) heart failure classification stage B. Patients without heart failure symptoms who have had an MI or who have evidence of left ventricular remodeling are thought to be at considerable risk of developing heart failure and intervention is warranted. Patients who are at risk of future heart failure should take an ACE inhibitor if they can tolerate it.In addition to optimal management of hyperlipidemia and hypertension, the AHA recommends that ACE inhibitors and β-blockers such as carvedilol, metoprolol succinate, or bisoprolol be used in all patients with a recent or remote history of MI, regardless of ejection fraction or the presence of heart failure (SOR A). Two large-scale studies have demonstrated that prolonged therapy with an ACE inhibitor reduces the risk of a major cardiovascular event even when treatment is initiated months or years after the MI.Furosemide is not recommended for use in stage B patients, and calcium channel blockers such as diltiazem can lead to worsening heart failure and should be avoided. The AHA recommends that angiotensin receptor blockers be administered to post-MI patients without heart failure who are intolerant of ACE inhibitors and have a low left ventricular ejection fraction (SOR B). Aldosterone antagonists would not be the first-line therapy for stage B heart failure.

A 78-year-old male comes to your office because he feels lightheaded and nauseated. His medical history includes heart failure and chronic atrial fibrillation, and he takes the following medications:Furosemide (Lasix)DigoxinQuinapril (Accupril)Warfarin (Coumadin)Atorvastatin (Lipitor)He recently took some ibuprofen after he injured his back while doing yard work. On examination his heart rate is 46 beats/min. He has a blood pressure of 110/70 mm Hg and a pulse rate of 48 beats/min while supine. After standing for 1 minute he has a blood pressure of 95/65 mm Hg and a pulse rate of 50 beats/min. After standing for 3 minutes he has a blood pressure of 104/60 mm Hg and a pulse rate of 50 beats/min. His hemoglobin level is 16.2 g/dL (N 14.0-18.0). A metabolic panel reveals a creatinine level of 1.2 mg/dL (N 0.6-1.5) and a potassium level of 4.3 mEq/L (N 3.5-5.5). His digoxin level is 1.8 ng/mL (therapeutic level 0.5-2.0).This patient's symptoms are most likely due to myopathy caused by atorvastatin toxicity digoxin toxicity gastritis from NSAID use chronic renal failure from NSAID use

B Digoxin has a very narrow therapeutic range and toxicity can occur even if serum levels are within this range. Levels may change drastically with use of NSAIDs, which reduce renal excretion of digoxin. Studies demonstrate that higher serum digoxin levels are associated with increased mortality and suggest that the effectiveness of digoxin therapy for heart failure in patients with a left ventricular ejection fraction ≤45% may be optimized by a serum digoxin level of 0.5-0.8 ng/mL. Elevated levels of digoxin can cause bradycardia or other serious arrhythmias and commonly cause nausea, vomiting, diarrhea, and dizziness/lightheadedness. Atorvastatin toxicity is manifested by myalgia and elevated creatine kinase, and rarely, rhabdomyolysis. Chronic renal failure and acute kidney injury may occur with use of NSAIDs, but the renal failure is unlikely to cause the bradycardia, and nausea is more common with severe end-stage renal failure. NSAID-related gastritis is unlikely to be the entire cause of his symptoms, given the reduced pulse rate.

Which one of the following cancer therapy drugs is a significant cause of heart failure? 5-Fluorouracil (5-FU) Doxorubicin Methotrexate Tamoxifen (Soltamox) Vinblastine

B Doxorubicin causes direct cardiotoxicity in a dose-related fashion, sometimes producing clinical heart failure. In one series, 5% of all patients taking doxorubicin developed clinical heart failure. Other chemotherapeutic agents can also cause heart failure and it is important to investigate past treatments that might be contributing to heart failure. The risk of doxorubicin-induced heart failure (which can occur within hours, weeks, or years after exposure) increases with the cumulative dose of anthracycline: 3%-5% with 400 mg/m2, 7%-26% at 550 mg/m2, and 18%-48% at 700 mg/m2.While tamoxifen increases the risk of thromboembolic events, it is not cardiotoxic. 5-Fluorouracil, methotrexate, tamoxifen, and vinblastine are not known contributors to heart failure.

Which one of the following is true regarding the use of aspirin in a patient undergoing coronary artery bypass graft (CABG) surgery? Aspirin should be withheld for 48 hours preoperatively to reduce bleeding complications Aspirin should be initiated 6 hours postoperatively if possible and continued indefinitely An aspirin dosage of 81 mg daily following CABG is sufficient to prevent vein graft closure Postoperative aspirin significantly reduces arterial graft closure during the first postoperative year

B In the past, concerns about bleeding complications limited the use of aspirin in the perioperative period in patients undergoing coronary artery bypass graft (CABG) surgery. Although associated with a modest increase in bleeding risk, aspirin given ≤24 hours before CABG has been shown to reduce postoperative morbidity and mortality and major cardiac and cerebral events (SOR B). Conversely, preoperative use of thienopyridines is associated with major bleeding complications, including pericardial tamponade, reoperation, and a greater need for transfusions, and should be discontinued prior to the procedure (clopidogrel and ticagrelor for 5 days, and prasugrel for 7 days) (SOR C).Although a beneficial effect on arterial graft patency has not been demonstrated, the early postoperative use of aspirin has been found to significantly reduce vein graft closure through the first postoperative year (SOR A). This benefit has been demonstrated with the initiation of aspirin 1, 7, and 24 hours after surgery, but is lost if aspirin is started more than 48 hours postoperatively. Current American Heart Association (AHA) guidelines recommend starting aspirin within 6 hours after surgery and continuing it indefinitely (SOR A). The American College of Cardiology, AHA, European Association for Cardio-Thoracic Surgery, and American College of Chest Physicians have issued guidelines recommending administration of early aspirin or an alternative (clopidogrel or ticlopidine) at 6 hours post CABG or soon after bleeding has settled as the standard of care for optimization of vein graft patency. Several randomized, controlled studies, including a meta-analysis, have shown that early administration of aspirin following CABG is not associated with increased blood loss or transfusion requirements.The Multicenter Study of Perioperative Ischemia Research Group found that in addition to the graft patency benefit, starting aspirin within 24 hours after surgery also reduced subsequent rates of mortality, myocardial infarction, stroke, renal failure, and bowel infarction. Although aspirin doses <100 mg daily have been shown to reduce adverse events in patients with coronary artery disease (CAD), they may be less efficacious than higher doses in maintaining saphenous vein graft patency. For this reason, AHA guidelines recommend a postoperative aspirin dosage of 100-325 mg daily (SOR C). Aspirin administration ≤24 hours before CABG was associated with a 75% reduction in 30-day mortality and a 57% reduction in major adverse cardiac and cerebrovascular events.

A 65-year-old female who had an ST-elevation myocardial infarction (STEMI) treated with fibrinolytic therapy 2 days ago has a sudden onset of chest pain and shortness of breath. Clinical evaluation reveals the presence of pulmonary edema with a blood pressure of 86/50 mm Hg, wet rales, a harsh holosystolic murmur along the left sternal border radiating toward the base and apex, and an S3 gallop. A pulmonary artery monitoring catheter is placed and oxygen saturation is found to be higher in the pulmonary artery than in the right atrium. Which one of the following complications does the patient most likely have? Acute papillary muscle rupture Ventricular septal rupture Ventricular free wall rupture Left ventricular aneurysm Severe left ventricular failure

B In the patient with an ST-elevation myocardial infarction (STEMI), cardiogenic shock should be considered if pulmonary edema and hypotension develop. Although extensive left ventricular dysfunction is responsible for 75% of cases, mechanical complications (e.g., acute, severe mitral regurgitation due to papillary muscle rupture, ventricular septal rupture, or subacute free wall rupture with tamponade) are another important cause. Conditions that can mimic cardiogenic shock include aortic dissection and hemorrhagic shock. Of the conditions listed, only ventricular septal rupture is associated with a pulmonary artery oxygenation that is higher than right atrial oxygenation.

A 62-year-old female sees you for follow-up of diabetes mellitus and hypertension. A drug-eluting stent was placed in her right coronary artery 2 years ago after an ST-elevation myocardial infarction. She has some dyspnea with walking up steep hills but can perform her usual household activities. Her current medications include the following:Lisinopril (Prinivil, Zestril), 20 mg dailyMetoprolol succinate (Toprol-XL), 100 mg dailyFurosemide (Lasix), 20 mg dailyMetformin (Glucophage), 1000 mg twice dailyInsulin glargine (Lantus), 35 U at bedtimeApixaban (Eliquis), 5 mg twice dailyOn examination her blood pressure is 128/74 mm Hg and her pulse rate is 70 beats/min and irregular. A cardiovascular examination reveals an irregularly irregular rhythm with no murmurs. Her lungs are clear. A recent echocardiogram showed an ejection fraction of 40%. Her current hemoglobin A1c is 7.7% and her estimated glomerular filtration rate is 47 mL/min/1.73 m2.Which one of the following would be most beneficial in terms of morbidity and mortality? Increasing metoprolol succinate to 150 mg daily Adding dapagliflozin (Farxiga), 10 mg daily Adding glimepiride (Amaryl), 4 mg daily Adding liraglutide (Victoza), 1.2 mg daily Adding pioglitazone (Actos), 15 mg daily

B Patients who have heart failure with reduced ejection fraction may benefit from dapagliflozin, which can reduce heart failure exacerbations, hospitalizations, and death. Pioglitazone can cause worsening of heart failure and should not be used. Glimepiride may improve this patient's hemoglobin A1c but has no beneficial effect on the heart, especially with her reduced ejection fraction. Liraglutide also may improve diabetic control and have some benefit for the treatment of developing ischemic heart disease, but it has not been proven to reduce hospitalizations for heart failure. Increasing the metoprolol dosage is also unlikely to provide additional benefit since the patient's resting pulse rate is 75 beats/min. Additional β-blockade may cause bradycardia and more symptoms.

A 55-year-old African-American male presents with dyspnea, small bilateral pleural effusions, and lower extremity edema. He has no history of elevated blood pressure or prior coronary artery disease. He reports drinking 2-3 home-brewed beers per week for the last 20 years. A chemistry profile and a CBC are normal. An EKG shows reduced voltage but is otherwise unremarkable. An echocardiogram shows increased wall thickening, with an ejection fraction of 55%.Which one of the following is the most likely cause of this patient's heart failure? Alcoholic cardiomyopathy Amyloid infiltration Hemochromatosis Hypertensive heart disease Ischemic heart disease

B The absence of a history of high blood pressure or anginal symptoms, along with an EKG that reveals no signs of left ventricular hypertrophy or ischemia, makes hypertensive and ischemic heart disease unlikely causes for this patient's heart failure. The lack of a significant history of excess alcohol use also makes alcoholic cardiomyopathy unlikely. Hereditary hemochromatosis is seen primarily in patients of Northern European descent. In patients with heart failure symptoms and reduced voltage on an EKG, the possibility of cardiac amyloidosis must be considered.Transthyretin-related cardiomyopathy (ATTR-CM) is a progressive infiltrative cardiomyopathy that mimics hypertensive, hypertrophic heart disease and often goes undiagnosed. In the United States the hereditary form disproportionately afflicts African-Americans, whereas wild-type ATTR-CM, a phenotypically similar condition, is more common in the white population. African-Americans tend to present with more advanced disease despite the fact that noninvasive genetic testing is available. This may be due to social determinants of health and a lack of access to specialized cardiac care.A cardiac biopsy and biopsies from other parts of the body are used to confirm the diagnosis. Nuclear scintigraphy is a noninvasive method that can detect ATTR-CM. Once a diagnosis of TTR-related cardiac amyloidosis is made, family members should be screened because early treatment is effective.

A 78-year-old male with chronic hypertension presents with a sudden onset of severe chest pain radiating to the back, associated with dyspnea and near-syncope. Which one of the following would suggest a diagnosis other than acute myocardial infarction? A 3/6 holosystolic apical murmur and diffuse ST-segment elevation A 2/6 diastolic murmur and weak radial and femoral pulses Diffuse ST-segment elevation of 1-2 mm A pulsus paradoxus of 10 mm Hg Chest and back pain that was mild initially and increased over the next 2 hours

B The chest pain of aortic dissection is typically described as searing, ripping, or tearing, and frequently radiates to the back or lower extremities. The pain is worst at the time of onset and lasts for hours. Helpful findings on physical examination include asymmetry of pulses or blood pressure, as well as a new murmur of aortic regurgitation (a decrescendo early diastolic murmur heard best in the aortic area, as opposed to holosystolic murmurs). This type of murmur indicates a dissection involving the ascending aorta. The dissection can extend to the pericardial sac and produce a pericardial friction rub on examination, as well as findings of cardiac tamponade. Pulsus paradoxus is a common finding of cardiac tamponade and is defined by a decrease in blood pressure of at least 12 mm Hg with inspiration.Aortic dissection is not usually associated with acute ischemic electrocardiographic changes. Data from the International Registry of Aortic Dissection indicates that ischemic changes were present on an EKG in only 15% of cases. The diagnosis can be established with transesophageal echocardiography, CT, or MRI. The importance of early diagnosis in a patient being evaluated for myocardial infarction is underscored by the fact that aortic dissection is exacerbated by fibrinolytic therapy and anticoagulation.Acute aortic dissection has a lethality rate of 1%-2% per hour after the onset of symptoms in untreated patients. Prompt diagnosis is therefore vital to increase the patient's chances of survival and prevent serious complications. Advanced age, male sex, a long-term history of arterial hypertension, and the presence of an aortic aneurysm confer the greatest population-attributable risk. However, patients with genetic connective tissue disorders such as Marfan, Loeys-Dietz, or Ehlers-Danlos syndrome, and patients with bicuspid aortic valves are at increased risk of aortic dissection at a much younger age.

A 59-year-old male sees you for a follow-up office visit after having a drug-eluting stent placed 6 weeks ago following a non-ST-elevation myocardial infarction. He also has a 2-year history of type 2 diabetes. He was discharged on the following medications:Aspirin, 81 mg dailyClopidogrel (Plavix), 75 mg dailyAtorvastatin (Lipitor), 40 mg dailyMetoprolol tartrate (Lopressor), 25 mg twice dailyRamipril (Altace), 10 mg dailyMetformin (Glucophage), 500 mg twice dailyThe patient has been asymptomatic since being discharged from the hospital. On examination he has a blood pressure of 142/86 mm Hg and a heart rate of 52 beats/min. The remainder of the examination is unremarkable. A lipid profile reveals an LDL-cholesterol level of 65 mg/dL, an HDL-cholesterol level of 30 mg/dL, and a serum triglyceride level of 260 mg/dL. His hemoglobin A1c is 7.2%.Which one of the following would be most appropriate at this time? Increasing the dosage of metoprolol Increasing the dosage of ramipril Adding gemfibrozil (Lopid) Adding niacin to increase HDL-cholesterol Switching from atorvastatin to rosuvastatin (Crestor), 20 mg daily

B The long-term management of patients with non-ST-elevation myocardial infarction involves measures to prevent recurrent cardiac events, as well as aggressive reduction of cardiovascular risk factors. The recommended pharmacotherapy to prevent death and myocardial infarction includes the following: aspirin, 81-325 mg daily, indefinitely adjunctive use of a P2Y12 inhibitor such as clopidogrel, 75 mg daily, or ticagrelor, 90 mg twice daily, for 6-12 months depending on bleeding risk, for all patients with non-ST-elevation acute coronary syndrome without contraindications β-blockers, with a target heart rate of 50-60 beats/min high-intensity statin therapy ACE inhibitors, particularly for patients with heart failure, left ventricular dysfunction, or diabetes mellitus Increasing the dosage of the ACE inhibitor is more appropriate than increasing the β-blocker dosage to improve this patient's blood pressure, given that his pulse rate is 52 beats/min.American Heart Association (AHA) guidelines no longer recommend the use of specific LDL-cholesterol targets. In patients who are 75 years of age or younger with clinical atherosclerotic cardiovascular disease, the 2018 AHA guidelines recommend high-intensity statin therapy be initiated or continued with the goal of a 50% or greater reduction in LDL-cholesterol. The guidelines specify high-intensity statin therapy as being either atorvastatin, 40-80 mg daily, or rosuvastatin, 20-40 mg daily.Patients should be educated about appropriate cholesterol management, blood pressure reduction, smoking cessation, and lifestyle management. Although the AHA guidelines for hypertension and coronary heart disease recommend a target blood pressure of <140/90 mm Hg in patients with acute coronary syndrome who are hemodynamically stable (SOR C), the AHA also considers a blood pressure target of <130/80 mm Hg at the time of hospital discharge a reasonable option (SOR C). The guidelines recommend that blood pressure be lowered slowly, however, and that decreases in diastolic pressure to <60 mm Hg be avoided since this could cause a potential reduction in coronary perfusion. Increasing ramipril for blood pressure control would thus be appropriate for a patient with bradycardia, up to the maximum dosage of 20 mg daily.Gemfibrozil should not be initiated in patients on statin therapy, as it increases the risk for muscle symptoms and rhabdomyolysis (SOR B). Fenofibrate may be considered concomitantly with a low- or moderate-intensity statin only if the benefits from cardiovascular risk reduction or lowering triglyceride levels that are >500 mg/dL are judged to outweigh the potential risk for adverse effects. The use of niacin might have been a consideration in the past in an effort to raise HDL-cholesterol and lower triglycerides, but support for its use was weakened by the findings of the AIM-HIGH trial (Atherothrombosis Intervention in Metabolic Syndrome with Low HDL/High Triglycerides: Impact on Global Health Outcomes), which found no clinical benefit from adding sustained-release niacin to a statin in patients with known coronary heart disease and low HDL-cholesterol.

In the United States, silent myocardial infarction is more common in which one of the following population groups? Women more than men Men more than women Hispanics more than non-Hispanic whites Non-Hispanic whites more than Asian-Americans

B There is no evidence that silent myocardial infarction (MI), as detected by the Minnesota code, is more common in women than in men in the ACCORD (Action to Control Cardiovascular Risk in Diabetes) trial cohort. Men were found to have a higher prevalence of silent MI on baseline EKGs than women (6% versus 4%, P = 0.001). Women had lower odds of silent MI than men after adjusting for other risk factors (odds ratio = 0.80, P = 0.04). Race and ethnicity were significantly associated with silent MI (P = 0.02), with Asian-Americans having the highest incidence and African-Americans and Hispanics having a lower incidence than non-Hispanic whites.

A 68-year-old male with a history of hypertension, diabetes mellitus, and heart failure presents with a 6-week history of progressive fatigue, ankle swelling, and dyspnea on exertion. His current medications include lisinopril (Prinivil, Zestril), 20 mg daily; atorvastatin (Lipitor), 40 mg daily; insulin glargine (Lantus), 10 U subcutaneously at bedtime; and sitagliptin (Januvia), 100 mg daily.On examination his pulse rate is 76 beats/min and regular, and his blood pressure is 130/80 mm Hg. He has jugular venous distention, a laterally displaced apex beat, and 1+ pitting ankle edema. Lung auscultation reveals bibasilar crackles. Cardiac auscultation reveals a regular rhythm with a soft S4. Echocardiography shows a left ventricular ejection fraction of 40%. A basic metabolic panel is normal, including a creatinine level of 1.1. mg/dL (N 0.7-1.3).Which one of the following should be started at this time? Carvedilol (Coreg), 12.5 mg twice daily Furosemide (Lasix), 20 mg twice daily Isosorbide dinitrate/hydralazine (BiDil), 20/37.5 mg three times daily Spironolactone (Aldactone), 25 mg twice daily

B This patient has signs of heart failure with fluid retention. Euvolemic status should be attained first in patients with fluid overload. Diuretics produce symptomatic benefits more rapidly than any other drug for heart failure and are the only agents that can adequately control fluid retention. Loop diuretics, such as furosemide, are more effective than thiazide diuretics for controlling sodium and free water clearance (SOR C).Although β-blockers should generally be prescribed for all patients with heart failure, they should not be started in patients with a current or recent history of fluid retention unless the patient is also on a diuretic. Furthermore, treatment with a β-blocker should be initiated at very low doses (e.g., carvedilol, 3.125 mg twice daily; metoprolol succinate extended release, 12.5-25.0 mg once daily) and gradually and cautiously increased as tolerated.Aldosterone antagonists are relatively weak diuretics that are prescribed to improve survival in selected patients with severe symptoms and a reduced left ventricular ejection fraction (SOR B).

A 57-year-old male is admitted to the hospital with an ST-elevation myocardial infarction and he undergoes successful percutaneous transluminal coronary angioplasty with placement of a new-generation drug-eluting stent. He does not have a history of bleeding. His medication regimen on discharge includes clopidogrel (Plavix) and aspirin.He should continue taking clopidogrel and remain on dual antiplatelet therapy for a minimum of 1 month 6 months 9 months 12 months

B in patients with coronary artery disease (CAD) who undergo percutaneous intervention (PCI), dual antiplatelet therapy (DAPT) is the standard initial strategy for antithrombotic therapy. Short-term DAPT for 1-3 months is feasible in most cases in the contemporary PCI era, but patients at a high ischemic risk, such as those presenting with acute coronary syndrome (ACS) and undergoing complex PCI, may benefit from long-term DAPT. Ischemic and bleeding risks should be comprehensively assessed using risk stratification models. Among the three currently available P2Y12 inhibitors, clopidogrel is the basic choice for treatment of stable CAD, while prasugrel and ticagrelor are more potent.A systematic review and meta-analysis has shown that long-term DAPT results in higher rates of bleeding and noncardiac mortality and standard-term DAPT was associated with higher rates of bleeding. In a subgroup analysis, long-term DAPT led to higher all-cause mortality than short-term DAPT in patients implanted with newer-generation drug-eluting stents (DES). Short-term DAPT has similar efficacy and safety compared to standard-term DAPT in patients presenting with ACS who received a newer-generation DES.DAPT with clopidogrel for 3-6 months has a similar efficacy and safety compared to 12 months of DAPT in patients treated with newer-generation DES and patients with ACS, without considering personal hemorrhagic profiles. Bleeding risk should be reduced by using a proton pump inhibitor during the DAPT period.

You see a 58-year-old male for a routine examination. According to the American College of Cardiology/American Heart Association classification system, which one of the following would meet the criteria for stage B heart failure, assuming he has no additional complications? A history of dyspnea on exertion Well compensated heart failure A grade 2/6 apical holosystolic murmur radiating to the axilla Uncontrolled type 2 diabetes

C A significant heart murmur, such as a grade 2/6 apical holosystolic murmur that radiates to the axilla, is generally meaningful. The American College of Cardiology/American Heart Association classification of heart failure includes four stages. Stage A is defined as the absence of structural disease in a patient at high risk for the development of heart failure. This includes patients with hypertension, atherosclerotic disease, diabetes mellitus, obesity, metabolic syndrome, or a family history of cardiomyopathy, as well as those using cardiotoxins. Patients with stage B heart failure have evidence of structural heart disease, such as a previous myocardial infarction, asymptomatic valvular disease, or evidence of left ventricular remodeling such as left ventricular hypertrophy or a low ejection fraction. Any patient with structural heart disease is at risk of heart failure and should be managed aggressively to prevent complications in the future.Stage C is defined as structural heart disease with prior or current symptoms of heart failure. Patients with stage D heart failure have refractory heart failure requiring specialized interventions.

A 68-year-old male with New York Heart Association class III heart failure with reduced ejection fraction and a blood pressure of 110/70 mm Hg is currently taking furosemide (Lasix), 40 mg twice daily, and carvedilol (Coreg), 12.5 mg twice daily. Which one of the following changes to this patient's current regimen will reduce his mortality risk and risk of future hospitalization for heart failure? Increasing the dosage of furosemide Adding digoxin Adding lisinopril (Prinivil, Zestril) Adding metolazone

C ACE inhibitors such as lisinopril have been shown to decrease both mortality and rehospitalizations for heart failure, and are the mainstay of treatment for patients who can take them. Digoxin improves symptoms and exercise tolerance but does not decrease mortality. There have been no long-term studies conducted to determine the effects of diuretics such as furosemide and metolazone on morbidity and mortality.

A patient presents with a 2-hour history of substernal chest pain. There is no ST-segment elevation on an EKG.Which one of the following new EKG findings would be an indication for instituting the same treatment protocol used for ST-elevation myocardial infarction? Atrial fibrillation Bifascicular block Left bundle branch block Right bundle branch block T-wave inversion

C According to the American College of Cardiology/American Heart Association guidelines, if there are no contraindications, fibrinolytic therapy or percutaneous coronary intervention should be administered to patients with an ST-elevation myocardial infarction (STEMI) whose symptoms began within the previous 12 hours and who have either ST elevation >0.1 mV in at least two contiguous precordial leads (or two adjacent limb leads) or new or presumably new left bundle branch block. In the absence of ST-segment elevation, new or presumably new left bundle branch block with symptoms of myocardial ischemia is a potential indication for using the treatment protocol for STEMI. Right bundle branch block, T-wave inversion, bifascicular block, and atrial fibrillation by themselves would not be indications for treatment using the STEMI protocol.

A 65-year-old African-American male presents with a 2-month history of exertional dyspnea and ankle swelling. His past medical history is notable for hypertension and angioedema related to a peanut allergy.On examination his blood pressure is 155/98 mm Hg. His jugular veins are mildly distended and bibasilar rales are noted. The cardiac examination reveals a regular rhythm with a soft S3 and no murmur. Examination of the lower extremities reveals 1+ pitting ankle edema. Echocardiography shows an estimated left ventricular ejection fraction of 40%.Which one of the following medications should be AVOIDED in this patient? Amlodipine (Norvasc) Carvedilol (Coreg) Enalapril (Vasotec) Furosemide (Lasix) Hydralazine

C Angioedema occurs in less than 1% of patients taking an ACE inhibitor but is more common in African-Americans. The American Heart Association recommends that ACE inhibitors not be initiated in any patient with a history of angioedema (SOR C). Calcium channel blockers, particularly those with negative inotropic effects such as verapamil and diltiazem, can cause a worsening of heart failure and should also be avoided (SOR C).Although angiotensin receptor blockers (ARBs) would be regarded as safe in this patient and may be considered as alternative therapy for patients who develop angioedema while taking an ACE inhibitor, angioedema can also occur in patients taking ARBs and extreme caution is advisable when substituting an ARB in a patient with a history of ACE inhibitor-associated angioedema (SOR C). There are no contraindications to the use of a diuretic or a β-blocker in this patient.

A 65-year-old female who has heart failure with an ejection fraction of 35% is found to have a TSH level of 13.8 µU/mL (N 0.3-4.82). Her T3 and T4 levels are normal, and her thyroid gland is normal to palpation. You check her levels again in 2 months and they are unchanged. You advise her that hypothyroidism decreases her metabolic rate, which reduces the stress on her heart hypothyroidism is detrimental to her heart only if she develops hypothyroid symptoms subclinical hypothyroidism has negative effects on heart failure and treatment should be considered treatment of subclinical hypothyroidism would raise her LDL-cholesterol level

C Clinical hypothyroidism has long been associated with cardiac dysfunction. It has also been shown that subclinical hypothyroidism (TSH >4 µU/mL with normal or borderline low thyroid hormone levels) can cause left ventricular systolic and diastolic dysfunction, which improves with thyroid replacement therapy. Patients with overt or subclinical hypothyroidism should be treated with levothyroxine to improve their cardiovascular function and decrease the potential risk of heart failure. Thyroxine in excess can exacerbate coronary artery disease, and should be started at low doses and increased slowly in patients with possible underlying coronary artery disease. Results of meta-analyses indicate that therapy will lower, not raise, serum LDL-cholesterol levels.

Which one of the following is true regarding the use of clopidogrel (Plavix) with aspirin in patients with coronary artery disease? A loading dose of 150 mg of clopidogrel is recommended at the time acute coronary syndrome is diagnosed Clopidogrel should be given first because it has a faster onset of antiplatelet activity Clopidogrel should be discontinued at least 5 days before coronary artery bypass graft surgery and aspirin should be continued up to the day of surgery When used with clopidogrel, aspirin can be given at a dosage of 325 mg daily after cardiac stent placement

C Clopidogrel should be discontinued at least 5 days before coronary bypass surgery but aspirin should be continued. Clopidogrel is a thienopyridine derivative that is used primarily as an adjunctive agent in patients with acute coronary syndrome (ACS). It is used most commonly in conjunction with aspirin but is an adequate alternative in patients who are aspirin intolerant. If clopidogrel is used alone, initial treatment with heparin or possibly with a glycoprotein IIb/IIIa inhibitor is especially important because of clopidogrel's delayed onset of antiplatelet activity compared to that of aspirin. The CAPRIE trial (Clopidogrel versus Aspirin in Patients at Risk of Ischaemic Events) found clopidogrel to be comparable to aspirin in reducing ischemic events in patients with a history of recent myocardial infarction, recent stroke, or symptomatic peripheral artery disease. The CURE trial (Clopidogrel in Unstable angina to prevent Recurrent Events) found the combination of aspirin and clopidogrel to be more effective in reducing ischemic events than aspirin alone in patients with ACS.Clopidogrel should be started with a loading dose of 300-600 mg, followed by 75 mg daily. When clopidogrel is used with aspirin, the aspirin dosage should be 75-162 mg daily. Because of an increased risk of bleeding, current guidelines recommend that clopidogrel be discontinued at least 5 days, and preferably 7 days, before bypass graft surgery. In patients undergoing urgent cardiac catheterization and percutaneous coronary intervention (PCI), a loading dose of 600 mg of clopidogrel should be administered either before or at the time of the PCI. Clopidogrel should be continued at a dosage of 75 mg daily, along with aspirin.

Which one of the following laboratory abnormalities is associated with structural heart disease and signs and symptoms of heart failure? Hypernatremia Elevated creatine kinase levels Elevated serum transaminases Low BNP levels Low circulating complement levels

C Hyponatremia and elevation of serum transaminases may occur with heart failure, due to chronic passive congestion of the liver and expansion of total body water under the influence of vasopressin. Elevation of creatinine kinase occurs in the setting of acute coronary syndrome but would not be expected in heart failure by itself. BNP is typically elevated in patients with symptomatic heart failure.

A 29-year-old male is evaluated in the emergency department for chest pain that started after he used cocaine, and which has now resolved. An EKG shows a prolonged QTc interval, new T-wave inversions, and biphasic T waves in leads V2 and V3. The physical examination reveals an anxious male with a blood pressure of 160/100 mm Hg and a heart rate of 118 beats/min.Which one of the following is true in this situation? The initial treatment should include aspirin and clopidogrel β-Blockers should never be used in patients with cocaine-related chest pain The treatment of choice is intravenous benzodiazepines and oral or intravenous nitrate therapy Nifedipine (Procardia) should be used as first-line therapy if the patient's blood pressure is elevated

C Myocardial infarction has been found in 6% of patients presenting to the emergency department with cocaine-associated chest pain. Cocaine precipitates coronary artery spasm by stimulating α-adrenergic receptors in smooth muscle cells in coronary arteries, as well as by increasing levels of endothelin-1 and reducing production of nitric oxide. Cocaine has also been found to increase the response of platelets to arachidonic acid, thus increasing thromboxane A2 production and platelet aggregation, and to lead to accelerated atherosclerosis in chronic users. Increased motor activity, along with skeletal muscle injury and rhabdomyolysis, is also associated with cocaine use, causing creatine kinase and even CK-MB elevation in the absence of myocardial infarction. As a result, cardiac troponin I or T is preferred for detecting myocardial necrosis.In most cases the initial management of cocaine-associated chest pain should include nitrate therapy and a benzodiazepine. Benzodiazepines relieve cocaine-associated chest pain and also reduce anxiety and the central stimulatory effects of the cocaine, indirectly improving hypertension and tachycardia. Nitrate therapy has been shown to reduce cocaine-associated chest pain, reverse cocaine-associated vasoconstriction, and lower blood pressure. Nifedipine is not recommended as first-line treatment but other calcium channel blockers such as diltiazem may be used for blood pressure control. It was previously thought that β-blockers were harmful in patients with chest pain associated with cocaine ingestion but that has been disproven in many recent studies. Labetalol and metoprolol are both safe and may have a beneficial effect in this situation.This patient's EKG is characteristic of Wellens syndrome, or left anterior descending coronary artery (LAD) T-wave syndrome. These EKG changes indicate post-ischemic reperfusion injury, typically related to critical narrowing of the LAD. Cocaine users may have EKG changes typical of Wellens syndrome as part of a vasospastic phenomenon without underlying stenosis.

You are considering adding a calcium channel blocker to the regimen of a patient who has hypertension and heart failure. Which one of the following is true regarding this class of drugs? Nondihydropyridine calcium channel blockers such as verapamil (Calan) and diltiazem (Cardizem) do not increase morbidity and mortality in patients with heart failure All calcium channel blockers can be used effectively for symptomatic control of angina in patients with heart failure Second-generation dihydropyridine calcium channel blockers such as amlodipine can be used safely for the treatment of hypertension in patients with heart failure Second-generation dihydropyridine calcium channel blockers often cause edema, which signals a significant exacerbation of heart failure

C Second-generation dihydropyridine calcium channel blockers such as amlodipine may be helpful in the management of refractory hypertension and angina in patients with concurrent heart failure (HF) who are on maximal doses of other recommended drugs. Amlodipine is generally well tolerated and has neutral effects on morbidity and mortality in large randomized, controlled trials. These drugs can be safely given with β-blockers in HF patients. Older nondihydropyridine calcium channel blockers can depress both conduction system function and ventricular function, and potentially worsen HF.From the standpoint of management of HF and decreasing HF-related morbidity and mortality, it makes sense to maximize dosages of drugs that have been shown to yield mortality benefit, including ACE inhibitors, angiotensin receptor blockers, β-blockers, and aldosterone antagonists, before adding calcium channel blockers. Second generation dihydropyridine calcium channel blockers such as amlodipine may have a greater role in patients with concurrent angina after the use of β-blocker therapy has been optimized, or for those in whom β-blockade is contraindicated.

A 74-year-old male with New York Heart Association class II heart failure and a left ventricular ejection fraction of 38% is on optimal dosages of an ACE inhibitor, a β-blocker, and pravastatin (Pravachol). His past medical history is unremarkable except for a long history of hypertension. He is a nonsmoker and reports that he has one glass of red wine with dinner every evening.On examination he has a blood pressure of 128/70 mm Hg and a BMI of 34 kg/m2. A lung examination is normal and a cardiac examination is notable for an S4 gallop.Which one of the following self-care measures should be recommended for this patient? Reducing sodium consumption to <1500 mg daily Reducing alcohol consumption to a weekly glass of wine Avoiding NSAID use Weight loss to attain a BMI <30 kg/m2

C Self-care is advocated as a method of improving outcomes in patients with heart failure. NSAIDs inhibit the synthesis of renal prostaglandins and can cause sodium and water retention, reducing the effectiveness of diuretics. Several observational cohort studies have revealed increased morbidity and mortality in patients with heart failure using either nonselective or selective NSAIDs. Available studies indicate that survival is highest in patients with a BMI of 30-32 kg/m2, and no studies have demonstrated a survival benefit from weight loss in patients with heart failure. Consequently, American Heart Association (AHA) guidelines currently recommend that weight loss be encouraged only in patients with a BMI >40 kg/m2.Several epidemiologic studies have failed to demonstrate a correlation between alcohol consumption and the development of heart failure. With the exception of patients with alcoholic cardiomyopathy, who should abstain from alcohol use, patients with heart failure who choose to drink should be advised to limit their alcohol intake to no more than 1-2 drinks a day.Several randomized, controlled trials that assessed outcomes with sodium restriction have all shown that lower sodium intake is associated with worse outcomes in patients with heart failure with reduced ejection fraction (HFrEF). These limitations make it difficult to give precise recommendations about daily sodium intake and whether it should vary with respect to the type of heart failure (HFrEF versus heart failure with preserved ejection fraction), disease severity such as New York Heart Association class, related comorbidities such as renal dysfunction, or other characteristics such as age or ethnicity. Because sodium intake is typically high (>4 g/d) in the general population, clinicians should consider some degree of sodium restriction, such as <3000 mg daily, to improve symptoms in patients with stage C or D heart failure.

63-year-old homeless male presents to the emergency department with a 1-week history of fatigue, a nonproductive cough, and intermittent chills. His medical history is significant for hypertension, chronic kidney disease, and gout. An EKG is normal but a cardiac troponin level is elevated.Which one of the following would be LEAST likely to cause the troponin elevation? Chronic kidney disease Compensated heart failure Gout Pneumonia Pulmonary embolism

C Since its introduction in the early 1990s, cardiac troponin (cTn) testing has emerged as a powerful tool for diagnosing myocardial infarction, as well as for risk stratification. While it is most often associated with ischemic events stemming from acute coronary syndrome (ACS), it is important to be aware that an elevated troponin level is simply a marker of cardiac myonecrosis and does not indicate the cause. The interpretation and clinical implications of an elevated troponin level depend on the clinical context in which the measurement was made.Most hospitals have now replaced conventional cTn tests with the new 5th generation high-sensitivity cTn (hs-cTn) T and I assays, which can detect troponin at concentrations 10- to 100-fold lower than conventional assays.To differentiate acute from chronic troponin elevation and to maintain a high specificity, clinical evaluation (pretest probability) and serial testing of hs-cTn are warranted. One of the various rule-in and rule-out algorithms should be followed when making a diagnosis of acute coronary syndrome. Elevated troponin results can be misinterpreted, and almost 13% of patients presenting with elevated hs-cTn levels and chest pain eventually prove not to have acute coronary syndrome.In addition, it is important to distinguish acute from chronic hs-cTn elevation. Acute causes of hs-cTn elevation are associated with a change in hs-cTn levels commonly referred to as the delta. Acute cardiomyocyte injury causes a steep release of troponins. Chronic, stable elevations of hs-cTn at or above the 99th percentile without a significant rise or fall can occur in patients with structural heart disease.Many nonischemic conditions are associated with elevated cardiac troponin levels, including heart failure, pulmonary embolism, chronic kidney disease, sepsis, chemotherapy-associated cardiac toxicity, myocarditis, severe acute neurologic disease such as stroke or subarachnoid hemorrhage, and infiltrative diseases such as amyloidosis and sarcoidosis. Gout should not elevate troponin levels. Unless there is a concern for ischemic heart disease, a cardiac troponin level should not be obtained, as there are many false-positive findings with hs-cTn assays.

A 63-year-old female has a previous history of hypertension and heart failure with reduced ejection fraction. You have treated her successfully with lisinopril (Prinivil, Zestril) in addition to other heart failure medications but she has developed severe angioedema, necessitating intubation.Of the following, which one would be the most appropriate combination that you could start today to control her blood pressure and treat her heart failure? Amlodipine (Norvasc) and furosemide (Lasix) Amlodipine and metoprolol tartrate (Lopressor) Hydralazine and isosorbide dinitrate Hydralazine and terazosin (Hytrin)

C The combination of nitrates and hydralazine has been shown to reduce mortality from heart failure. The data supporting a benefit from ACE inhibitors and angiotensin receptor blockers (ARBs) is much more robust, so the nitrate and hydralazine combination is most appropriate for those patients who are intolerant of ACE inhibitors or ARBs, or in whom these medications are contraindicated. One trial limited to African-American patients demonstrated benefit from the nitrate and hydralazine combination when added to therapy with an ACE inhibitor and/or a β-blocker. Hydralazine and isosorbide dinitrate combinations are underutilized and could provide significant clinical benefit to many patients with heart failure who are unable to tolerate other drug regimens.α-Blockers and calcium channel blockers are not indicated for the treatment of heart failure unless added blood pressure control is needed despite maximally tolerated standard heart failure medications. Losartan or other ARBs should not be given to patients who develop severe angioedema while taking an ACE inhibitor unless there is a washout period of several weeks and only then with very careful monitoring.

An 82-year-old female presents with increasing dyspnea. Her husband is worried because she occasionally stops breathing when she is asleep. You have been treating the patient for heart failure for the past 2 years with ACE inhibitors, β-blockers, diuretics, and low-dose spironolactone (Aldactone). The nurse who measures the patient's blood pressure notes that the systolic sounds are heard first at a pressure of 135 mm Hg and a pulse rate of 40 beats/min. At 120 mm Hg the nurse hears Korotkoff sounds at a regular rate of 80/min.Which one of the following is true regarding this patient? The examination findings are normal for patients in this age group The patient's breathing pattern is normal for patients in this age group Both the breathing and blood pressure findings may improve with more intensive treatment Medications should be reduced in this patient because her blood pressure is unstable

C This patient has pulsus alternans, which is common in patients with decompensated heart failure and advanced myocardial disease. Effective treatment can make this finding disappear. Cheyne-Stokes breathing is also common in patients with decompensated heart failure. If the heart failure is treated, the breathing abnormality can disappear. The patient has symptomatic heart failure, which classifies her heart failure as stage C at least, according to the American College of Cardiology/American Heart Association heart failure guidelines.

An 84-year-old female presents with the new onset of mild symmetric bilateral leg swelling and exertional dyspnea. She has been treated for hypertension for many years with amlodipine (Norvasc) and hydrochlorothiazide. She has a history of smoking for many years but quit 30 years ago. On examination her blood pressure is 160/75 mm Hg. A laboratory evaluation rules out thyroid disease and anemia. Her estimated glomerular filtration rate is 47 mL/min/1.73 m2. Her BNP level is 350 pg/mL (N <100). Echocardiography reveals an ejection fraction of 55%, and a chest film shows pulmonary vascular congestion and small pleural effusions with a normal-sized heart and no other abnormal findings.Based on this presentation, which one of the following is the most likely diagnosis? A side effect of amlodipine COPD Heart failure Venous insufficiency edema

C This patient most likely has heart failure with preserved ejection fraction (HFpEF), which is characterized by a normal ejection fraction (>50%) and clinical findings of heart failure. Her ejection fraction of 55% rules out systolic dysfunction. Long-standing hypertension, especially with elevated pulse pressure, is a cause of HFpEF. COPD is unlikely to present for the first time 30 years after smoking cessation. Venous insufficiency and leg edema may result from amlodipine use, but they would not present in a sudden fashion.

A 73-year-old male sees you for routine follow-up. He has a history of non-ST-elevation myocardial infarction and placement of a stent in the right coronary artery 4 years ago. He also has hyperlipidemia treated with a high-intensity statin and gout treated with allopurinol (Zyloprim). His blood pressure is elevated on three separate readings to an average of 142/85 mm Hg. A basic metabolic panel is normal.Which one of the following would be the best choice for improving blood pressure and reducing all-cause mortality in this patient? Amlodipine (Norvasc) Atenolol Hydrochlorothiazide Lisinopril (Prinivil, Zestril)

D ACE inhibitors are an excellent choice for reducing heart disease and other secondary outcomes such as stroke and all-cause mortality. Blood pressure should be reduced to a goal of <130/80 mm Hg in patients with known heart disease. Thiazide diuretics could precipitate a gout attack in the future by increasing the uric acid. β-Blockers such as carvedilol, metoprolol tartrate, metoprolol succinate, nadolol, bisoprolol, propranolol, and timolol can be used in many circumstances. Despite the fact that β-blockers are effective for improving outcomes post myocardial infarction, they tend to lose some of their effectiveness in older patients. Because of their intrinsic sympathomimetic activity, β-blockers should generally be avoided in these patients. Atenolol in particular should not be used, because it is less effective than placebo in reducing cardiovascular events. Dihydropyridine calcium channel blockers reduce blood pressure and are also associated with improved cardiac outcomes, but do not reduce all-cause mortality as well as ACE inhibitors.

A 61-year-old male is being treated for erectile dysfunction with intermittent tadalafil (Cialis), 20 mg. He calls 911 due to severe chest pain.It would not be considered safe to administer nitrate therapy unless the time interval since his last dose of tadalafil has been a minimum of 6 hours 12 hours 24 hours 48 hours 96 hours

D Administration of nitrates in close proximity to the use of a phosphodiesterase-5 (PDE-5) inhibitor has been associated with profound hypotension, myocardial infarction, and even death. Nitrates should not be administered to patients who have taken sildenafil or vardenafil within the previous 24 hours. The time interval should be 48 hours for patients who have taken tadalafil because it has a half-life of 17.5 hours, as opposed to approximately 4 hours for sildenafil and vardenafil.

A 69-year-old female with a history of chronic hypertension and a previous myocardial infarction sees you for follow-up 6 weeks after being hospitalized for chest pain. During her hospitalization she underwent cardiac catheterization, which showed only a lesion in the circumflex that was less than 50% occluded. An EKG revealed sinus bradycardia of 52 beats/min, multifocal PVCs, and a QRS interval of 0.10 sec. Echocardiography revealed a left ventricular ejection fraction of 32%.Although the patient feels comfortable at rest she reports that she has difficulty walking up a single flight of stairs. Her current medications include atorvastatin (Lipitor), 40 mg daily; lisinopril (Prinivil, Zestril), 20 mg daily; metoprolol succinate (Toprol-XL), 100 mg daily; furosemide (Lasix), 40 mg daily; and aspirin, 81 mg daily.On examination the patient is not in acute distress. Her blood pressure is 132/78 mm Hg and her pulse rate is 55 beats/min. A lung examination reveals bibasilar rales. Auscultation of the heart reveals a regular rhythm with a soft S3 and S4 and no murmur.Which one of the following interventions has been shown to improve survival in patients such as this? Increasing the furosemide dosage Adding amlodipine (Norvasc) Adding digoxin Adding eplerenone (Inspra) Cardiac resynchronization therapy

D Aldosterone antagonists are important in the management of severe heart failure. The addition of an aldosterone antagonist to a β-blocker and an ACE inhibitor was shown in the Randomized Aldactone Evaluation Study to reduce rates of death and hospital readmissions in selected patients with moderate to severe symptoms of heart failure and a reduced left ventricular ejection fraction (LVEF) (SOR B). More recently, the EMPHASIS-HF trial (Eplerenone in Mild Patients Hospitalization and Survival Study in Heart Failure trial) found that the addition of eplerenone in heart failure patients with mild symptoms consistent with New York Heart Association (NYHA) class II heart failure and a mean LVEF of 26% resulted in a reduction in both hospitalizations and deaths. Current American Heart Association guidelines recommend the addition of an aldosterone antagonist to an ACE inhibitor and a β-blocker in selected patients with moderately severe to severe symptoms of heart failure and a reduced LVEF.Although the addition of digoxin can be of benefit in selected heart failure patients by reducing the risk for hospitalization, it has not been shown to reduce mortality (SOR B). According to recent guidelines, patients are considered candidates for cardiac resynchronization therapy if they have NYHA class II-IV heart failure, a left ventricular ejection fraction ≤35%, and a QRS duration >130 ms on an EKG. However, 30%-35% of patients who meet these criteria are nonresponders with no symptomatic improvement or reverse left ventricular remodeling. Left bundle branch block morphology, a QRS duration ≥150 ms, and adequate coronary sinus anatomy have been most closely associated with a favorable response. Mitral valve regurgitation, right ventricular dysfunction, and atrial fibrillation have been shown to have a negative impact on patient response. Calcium channel blockers can lead to worsening heart failure and an increased risk of cardiovascular events and should be avoided.

You admit a patient with acute coronary syndrome to the hospital. Which one of the following is true regarding the differences between low molecular weight heparin (LMWH) and unfractionated heparin (UFH) in this situation? The use of glycoprotein IIb/IIIa inhibitors does not require a change in the dosage of UFH The dosage of both should be titrated to achieve a partial thromboplastin time of 1.5-2.5 times control Platelet activation is the same for both The incidence of thrombocytopenia is lower with LMWH UFH has higher bioavailability because it is given intravenously

D Anticoagulation is recommended in addition to antiplatelet therapy for all patients with acute coronary syndrome regardless of the initial treatment strategy. For patients managed with an early invasive strategy, heparin exerts its anticoagulant effect by accelerating the action of circulating antithrombin. It is available as either intravenous unfractionated heparin (UFH) or subcutaneous low molecular weight heparin (LMWH).LMWH offers greater bioavailability than UFH because of decreased binding to plasma proteins and endothelial cells, and it results in less platelet activation. The incidence of thrombocytopenia in patients treated with LMWH is less than with UFH. LMWH does not change the partial thromboplastin time (PTT) appreciably, so PTT should not be used to monitor the dosage. LMWH is a viable option for treatment of acute coronary artery syndrome and is preferred in many situations.If UFH is used it should be given intravenously at a dosage of 85 U/kg unless a glycoprotein IIb/IIIa inhibitor is also administered, in which case the dosage should be reduced to 60 U/kg. Dosing adjustments should be based on the target activated clotting time. Patients treated with UFH should be monitored by factor Xa assays.

According to National Cholesterol Education Program guidelines, which one of the following is a coronary heart disease (CHD) risk equivalent? A CHD risk score of 15% A strong family history of heart disease A first degree male relative who had a myocardial infarction before age 45 Peripheral artery disease Uncontrolled hypertension

D As defined by the Adult Treatment Panel III of the National Cholesterol Education Program (NCEP), coronary heart disease (CHD) equivalents are risk factors for major coronary events equal to established CHD (>20% risk for CHD over 10 years). CHD risk equivalents include other clinical forms of atherosclerotic disease such as peripheral artery disease (PAD), abdominal aortic aneurysm, and symptomatic carotid artery disease; diabetes mellitus; and combinations of multiple risk factors that confer a 10-year risk for CHD exceeding 20%. Hypertension alone or a family history alone are not defined as CHD risk equivalents in the NCEP report.Patients with these CHD equivalents are included in the statin benefit groups identified by the 2013 American College of Cardiology/American Heart Association cholesterol guidelines. High-intensity statin therapy is recommended for patients with clinical atherosclerotic cardiovascular disease, including acute coronary syndrome; a past history of myocardial infarction, stable or unstable angina, or coronary or other arterial revascularization; or a history of stroke, TIA, or peripheral artery disease presumed to be of atherosclerotic origin.

A 61-year-old male sees you for a follow-up visit. His medical history includes end-stage heart failure, chronic atrial fibrillation, a left ventricular ejection fraction of 30%, and stage 4 chronic kidney disease. He is taking optimal dosages of lisinopril (Prinivil, Zestril), metoprolol succinate (Toprol-XL), furosemide (Lasix), digoxin, and spironolactone (Aldactone). He continues to have symptoms of heart failure with minimal exertion, but not at rest. An EKG shows a ventricular rate of 85 beats/min, a QRS duration of 0.14 sec, and old Q waves in the inferior leads.Appropriate management options for this patient include which one of the following? Adding a nondihydropyridine calcium channel blocker Adding a thiazide diuretic Switching from metoprolol succinate to metoprolol tartrate (Lopressor) Synchronized biventricular pacing

D Biventricular pacing with an implantable defibrillator can improve symptoms and increase survival in heart failure patients with a prolonged QRS duration, and is recommended for those with a low ejection fraction, given their increased risk for ventricular fibrillation.Patients with refractory heart failure on optimal medical therapy should be considered for a heart transplant. Patients with an anticipated 1-year survival probability <50% can benefit from left ventricular (LV) assist devices. Patients who have a narrow QRS and stage D heart failure despite optimal medical therapy, and who are not candidates for transplant or LV assist devices, should not receive a defibrillator if their expected survival related to heart failure or other comorbidities is less than 1-2 years, since a defibrillator will not improve their survival.Changing from metoprolol succinate to metoprolol tartrate will not be beneficial since the succinate form is the preferred formulation for heart failure. Nondihydropyridine calcium channel blockers reduce the ejection fraction and would therefore not be beneficial in this patient. Patients with severe heart failure and severe chronic kidney disease generally do not respond favorably to thiazide diuretics.

A 62-year-old male comes to your office for a routine health maintenance evaluation. He has a history of hypertension, type 2 diabetes, and New York Heart Association class II heart failure. His current medications include metformin (Glucophage), 500 mg twice daily; benazepril (Lotensin), 40 mg daily; chlorthalidone, 12.5 mg daily; atorvastatin (Lipitor), 10 mg daily; and aspirin, 81 mg daily. A physical examination is notable only for a BMI of 29 kg/m2 and a blood pressure of 135/80 mm Hg. His hemoglobin A1c is 6.9%.Which one of the following additional medications would be appropriate to help manage his heart failure? Amlodipine (Norvasc) Digoxin Losartan (Cozaar) Metoprolol succinate (Toprol-XL) Metoprolol tartrate (Lopressor)

D Current American Heart Association guidelines recommend that a β-blocker, specifically either carvedilol, bisoprolol, or metoprolol succinate, be prescribed to all patients with stable heart failure with a reduced left ventricular ejection fraction. These three β-blockers have all been shown to prolong survival in patients with current or prior symptoms of heart failure. A class effect cannot be assumed. Studies have shown short-acting metoprolol tartrate to be less effective than sustained-release metoprolol succinate in reducing the risk of death in patients with chronic heart failure. Losartan should not be added to an ACE inhibitor. Amlodipine adds no benefit for heart failure. Digoxin would not be indicated in this patient since there is no history of atrial fibrillation or other tachyarrhythmia.

Coronary atherosclerotic plaques that are prone to disruption, thereby causing acute coronary syndrome, are characterized by which one of the following? The absence of inflammatory cells An abundance of smooth muscle cells A "hard," fibrous, lipid-poor core Development at bends or branch points in the arterial tree

D Disrupted plaques are found beneath 75% of the thrombi responsible for acute coronary syndromes. Plaques that are prone to disruption are usually nonobstructive and are often located at branch points or bends in the arterial tree. They are characterized histologically by abundant numbers of macrophages and inflammatory cells. The atheromatous core of a disrupted plaque is generally soft, with a consistency like that of toothpaste. Collagen is important to the tensile strength of tissue, and a reduced amount of collagen and a relative lack of smooth muscle cells is typical in disrupted plaques.

A 54-year-old male with a history of type 2 diabetes and psoriasis presents to your clinic for routine follow-up. Which one of the following laboratory findings is associated with an increased risk of cardiovascular disease? Low plasma homocysteine levels A low serum fibrinogen level Elevated adiponectin levels A urine albumin/creatinine ratio of 57

D Elevated levels of C-reactive protein, serum fibrinogen, lipoprotein(a), and plasma homocysteine, as well as an increased urine albumin/creatinine ratio, are associated with an increased risk for atherosclerotic vascular disease. At this time, however, routine measurement of fibrinogen, homocysteine, or apolipoprotein levels is not recommended for the diagnosis of coronary artery disease.Adiponectin, a protein hormone secreted predominantly by adipocytes, improves insulin sensitivity and blood lipid levels, and lowers inflammatory cardiovascular risk factors. In the Health Professionals Follow-up Study, high plasma adiponectin levels were associated with a lower risk of myocardial infarction in men.Patients with chronic kidney disease or inflammatory rheumatologic conditions have an increased risk of cardiovascular disease.

Which statin option is recommended for patients who have been diagnosed with acute coronary syndrome? Atorvastatin (Lipitor), 20 mg daily Lovastatin, 40 mg daily Pravastatin (Pravachol), 80 mg daily Rosuvastatin (Crestor), 20 mg daily Simvastatin (Zocor), 40 mg daily

D High-intensity statins are recommended in patients with acute coronary syndrome (ACS). According to the American College of Cardiology/American Heart Association cholesterol guidelines, atorvastatin, 40-80 mg daily, and rosuvastatin, 20-40 mg daily, are considered high-intensity statin therapy. Moderate-intensity therapy lowers the LDL-cholesterol (LDL-C) level by 30%-50% and includes pravastatin, 40-80 mg daily; lovastatin, 40 mg daily; atorvastatin, 20 mg daily; and simvastatin, 20-40 mg daily.The 2018 guidelines state that the greater the reduction in LDL-C from statin therapy, the greater the risk reduction. For example, lowering LDL-C levels by 1% results in an approximately 1% reduction in the risk of atherosclerotic cardiovascular disease (ASCVD). The risk reduction is somewhat greater at higher baseline LDL-C levels and somewhat less at lower baseline levels. A specific LDL level is not recommended except in patients with very high-risk features. Instead, they recommend a decrease in LDL of at least 50% for most patients.Patients at very high risk include those with a history of multiple major ASCVD events or one major ASCVD event and multiple high-risk conditions. For this group of patients the guidelines recommend an LDL-C threshold of 70 mg/dL. If this cannot be obtained by maximally tolerated statin therapy, the addition of nonstatin medications should be considered. Ezetimibe may be added to maximally tolerated statin therapy initially, and if the LDL-C level is still above the target level, adding a PCSK9 inhibitor such as alirocumab or evolocumab may be considered.

A 76-year-old female sees you for follow-up 2 weeks after she was hospitalized for heart failure. Her past medical history is notable for heart failure, hypertension, coronary heart disease, and well controlled depression. She does not smoke. Her current medications include the following:Lisinopril (Prinivil, Zestril), 20 mg dailyHydrochlorothiazide, 25 mg dailyFurosemide (Lasix), 40 mg dailyMetoprolol succinate (Toprol-XL), 50 mg dailyMetformin (Glucophage), 850 mg twice dailySimvastatin (Zocor), 40 mg dailyCitalopram (Celexa), 20 mg dailyAspirin, 81 mg dailyOn examination, the patient is afebrile, her blood pressure is 130/82 mm Hg, her pulse rate is 90 beats/min, and her respiratory rate is 20/min. Her jugular veins are mildly distended. Examination of the lungs reveals bibasilar crackles. The cardiac examination reveals a regular rhythm, an S4 gallop, and no murmurs. She has 1+ bilateral edema to the shins. A laboratory evaluation is notable only for a serum sodium level of 122 mEq/L (N 135-145).Which one of the following would be appropriate for managing her hyponatremia? Increasing her salt intake Increasing furosemide Increasing lisinopril Discontinuing citalopram Prescribing low-dose desmopressin

D Hyponatremia is a common problem in patients with heart failure, and its severity correlates directly with the degree of myocardial dysfunction. Hypervolemic hyponatremia is the type most commonly associated with heart failure, with edema indicating increased total body sodium and water. Heart failure is associated with inappropriately elevated plasma arginine vasopressin levels, which causes impaired water excretion, a dilutional hyponatremia, and increased ventricular preload. Management generally calls for a reduction in water intake and improving cardiac function.All SSRIs such as citalopram are associated with a high incidence of hyponatremia, and elderly patients may be at increased risk for this side effect. Physicians caring for elderly patients should be aware of this potentially serious but reversible adverse effect.Thiazide diuretics are associated with impaired renal water excretion, and reducing the dosage of thiazide diuretics or discontinuing their use is recommended. Although increasing sodium and water intake is the primary treatment for hypovolemic hyponatremia, patients with heart failure do not benefit from this strategy. Desmopressin is a vasopressin analog and is contraindicated in patients with hyponatremia. Arginine vasopressin antagonists, including tolvaptan and conivaptan, can be considered for patients with severe or recalcitrant hyponatremia.

A 65-year-old female has a history of long-standing hypertension that has been treated only intermittently due to a lack of insurance. She was diagnosed with type 2 diabetes 2 months ago at her first health care visit in 2 years. She sees you today with exertional dyspnea and lower extremity edema which have been present for the past month. She is found to be in heart failure, with mild bibasilar rales. An EKG is normal but echocardiography reveals global hypokinesis with an ejection fraction of 40%. She responds well to initial diuretic therapy and ACE inhibitor therapy but at a follow-up visit 1 week later she reports chest heaviness from walking about one block, which is relieved by rest. A resting EKG shows no change from the previous EKG.The most appropriate next step in the workup of this patient would be a treadmill stress EKG treadmill stress echocardiography a dipyridamole technetium-sestamibi stress test cardiac catheterization

D In patients with left ventricular dysfunction and angina the pretest probability of coronary artery disease is high enough to make nonpharmacologic or pharmacologic stress tests of minimal utility. Coronary angiography, and revascularization if indicated, is the appropriate next step.

A 74-year-old female is discharged from the hospital after being treated for an exacerbation of heart failure with volume overload. She has a previous history of coronary heart disease and hypertension. Her discharge medications include furosemide (Lasix), 20 mg twice daily; lovastatin, 40 mg daily; ramipril (Altace), 5 mg daily; spironolactone (Aldactone), 12.5 mg twice daily; metoprolol succinate (Toprol-XL), 75 mg daily; and aspirin, 81 mg daily. In addition, she is instructed to avoid the use of ibuprofen and other NSAIDs and to add metolazone, 2.5 mg daily, with 30 mL of 10% potassium chloride elixir on mornings when her weight is more than 3 lb over her target weight of 130 lb.Which one of the following is the most common reason for medication nonadherence in patients such as this? Cost Concerns regarding potential side effects Conflicting instructions from different health care providers Failure to understand discharge instructions Doubts about the need for the medications

D Medication compliance and understanding of how and why to take medications is a crucial aspect of medical care in heart failure. A study of patients recently discharged from the hospital following an exacerbation of heart failure found a high rate of medication nonadherence, with only one-third of patients taking all their medications as prescribed and not taking unprescribed medications. Of those not taking medications as prescribed, the most common reason given was not understanding discharge instructions (57%). Less common reasons include confusion due to conflicting instructions from the discharging physician and the primary care physician, medication cost, being unconvinced of the utility of the medication, and concerns regarding potential side effects (SOR B).

A 63-year-old male was recently diagnosed with moderate obstructive sleep apnea. He reports daytime drowsiness and says that he has become progressively more fatigued. He has known heart failure that has resulted in three prior hospital admissions, but he has remained stable over the past 6 months. He asks you about continuous positive airway pressure (CPAP) therapy.Which one of the following would be the most appropriate advice regarding CPAP therapy for this patient? It is associated with a significant risk of arrhythmia in patients with heart failure It is associated with a significant risk of increased edema in patients with heart failure It will have no effect on his heart failure but is indicated to help his sleep apnea symptoms It will likely improve his heart function in addition to improving symptoms related to his sleep apnea

D Sleep apnea is an important commonly unrecognized cause of heart failure. Continuous positive airway pressure (CPAP) therapy has been shown to improve a number of cardiovascular parameters. In a placebo-controlled trial, CPAP therapy in patients with heart failure resulted in improvements in blood pressure, heart rate, and ejection fraction after only 1 month of therapy. Long-term patient outcome studies have not been published. Epidemiologic research indicates that obstructive sleep apnea is associated with increases in the incidence and progression of coronary heart disease, heart failure, stroke, and atrial fibrillation.

Which one of the following is true regarding statin-related myalgia? A normal serum creatine kinase level excludes the diagnosis Serum creatine kinase levels of 3-4 times the upper limit of normal is a reason to discontinue statin therapy Grapefruit juice intake carries an equal risk for statin-associated myopathy with either simvastatin (Zocor) or pravastatin (Pravachol) Concomitant use of diltiazem (Cardizem) increases the risk of myopathy in patients treated with simvastatin Combination therapy with a statin and fenofibrate (Tricor) increases the risk for myopathy

D Statin-related muscle complaints include statin-associated muscle symptoms (any muscle complaints occurring in patients taking a statin, but not necessarily caused by the statin), myalgia (muscle complaints without serum creatine kinase [CK] elevations), myopathy (unexplained muscle pain or weakness accompanied by a CK concentration >10 times the upper limit of normal), and associated renal failure. The risk of statin-induced serious muscle injury, including rhabdomyolysis, is <0.1%, and the risk of serious hepatotoxicity is approximately 0.001%. The risk of rhabdomyolysis and other adverse effects related to statin use can be exacerbated by several factors, including compromised hepatic and renal function, hypothyroidism, diabetes mellitus, and concomitant medication use.Simvastatin, atorvastatin, and lovastatin are primarily metabolized by the cytochrome P-450 (CYP) 3A4 system. Medications that inhibit CYP 3A4, such as macrolide antibiotics and azole antifungals, increase the serum concentrations of these statins and therefore the risk of statin-associated myopathy. Other medications that can increase the risk of statin-associated myopathy include fibric acid derivatives such as gemfibrozil, niacin, HIV protease inhibitors, nefazodone, verapamil, diltiazem, and amiodarone. Azalide antibiotics such as azithromycin have minimal effects on statin pharmacokinetics. Fenofibrate has little, if any, potential to cause myopathy/rhabdomyolysis when given alone or with a statin, and has no clinically important effect on statin pharmacokinetics.Grapefruit juice also affects the metabolism of many medications, particularly most statins, through the inhibition of the CYP 3A4 pathway. Levels of lovastatin and simvastatin are most affected and may increase by 100%-250% with intake of grapefruit juice. Pravastatin plasma concentrations are not significantly affected by any CYP inhibition and only slightly affected by inducers. Daily consumption of only 300 mL of grapefruit juice has a minimal effect on the pharmacokinetics of atorvastatin. Combination therapy with ezetimibe has not been shown to increase the risk of myopathy.Although some experts recommend baseline serum CK measurements prior to the start of statin therapy, routine monitoring of CK levels is not required. There is no need to discontinue statin therapy in asymptomatic patients unless CK levels are more than 10 times higher than the upper limits of normal.

Cardioselective β-blockers are contraindicated in patients with which one of the following? Moderate persistent asthma Hypertension Hyperthyroidism Raynaud's phenomenon Supraventricular tachycardia

D β-Blockers are helpful for controlling symptoms of hyperthyroidism and the ventricular rate in patients with supraventricular tachycardia if they are not complicated by other conditions. Although recent research is controversial, β-blockers can increase the frequency of symptoms in patients with Raynaud's phenomenon and generally should be avoided unless absolutely needed for other concomitant medical conditions.Current data suggests that the risk of asthma being worsened by systemic nonselective β-blockers outweighs any potential benefits they may have for other clinical problems. Recent studies have shown that systemic cardioselective β-blockers, on the other hand, are not associated with a significant increased risk of moderate or severe asthma exacerbations. Nonselective β-blockers should not be prescribed for patients with asthma, but cardioselective β-blockers, preferably in low doses, may be used when strongly indicated and other therapeutic options are not available.

A 57-year-old male with a history of chronic stable angina and type 2 diabetes presents with a recent increase in symptoms. An EKG is notable for the presence of first degree AV block and left anterior hemiblock. Coronary angiography reveals three-vessel disease with a left ventricular ejection fraction of 45%.Which one of the following interventions would offer the greatest survival benefit? Intensive medical management A permanent pacemaker An implantable cardiac defibrillator Percutaneous coronary intervention Coronary artery bypass graft surgery

E Angiographic characteristics of high-risk groups with improved survival after surgical management include left main coronary artery stenosis, three-vessel disease with a left ventricular ejection fraction <50%, and two- or three-vessel disease with >75% stenosis of the proximal left anterior descending artery (LAD).A meta-analysis of three major trials confirmed the 10-year survival benefit from surgery for patients with three-vessel disease, two-vessel disease, and single-vessel disease that included stenosis of the proximal LAD, regardless of whether the patient had a normal or abnormal left ventricular ejection fraction. Large randomized trials that have reached 7-8 years of follow-up have generally shown that survival for patients with diabetes mellitus is better with coronary artery bypass (CABG) surgery than with percutaneous coronary intervention (PCI). The patient described does not have an indication for the placement of either an implantable cardiac defibrillator or a pacemaker, given that his ejection fraction is >35% and he does not have complete heart block.Patients who have diabetes with significant two- or three-vessel disease or those with single-vessel proximal LAD or left main disease generally do better with coronary artery bypass than with percutaneous intervention. A study of 3131 patients showed that at 5 years or the longest follow-up, patients with diabetes randomized to CABG had a lower all-cause mortality rate than those randomized to PCI with either a drug-eluting stent or a bare metal stent (relative risk = 0.67; P = 0.002). There is a higher risk of stroke with CABG than with PCI.

At a routine follow-up visit a 62-year-old male with a history of coronary artery disease and heart failure reports an increase in dyspnea with moderate exertion. His current medications are enalapril (Vasotec), 30 mg daily; carvedilol (Coreg), 12.5 mg twice daily; spironolactone (Aldactone), 25 mg daily; furosemide (Lasix), 40 mg daily; and aspirin, 81 mg daily. A physical examination is notable for a blood pressure of 128/82 mm Hg and a heart rate of 62 beats/min. There are no crackles or edema noted. Echocardiography during a hospitalization 4 months ago revealed a left ventricular ejection fraction of 35%. A CBC and metabolic panel today are normal. A BNP level is 250 pg/mL (N <100).Which one of the following changes should you make to this patient's medication regimen? Increase furosemide to 40 mg twice daily Add ivabradine (Corlanor), 5 mg twice daily Add losartan (Cozaar), 50 mg daily Discontinue enalapril and make no additional changes Discontinue enalapril for 2 days and add sacubitril/valsartan (Entresto), 49/51 mg twice daily

E Randomized, controlled trials have clearly established the benefit of using an evidence-based β-blocker such as carvedilol, metoprolol succinate, or bisoprolol in conjunction with either an ACE inhibitor or an angiotensin receptor blocker (ARB) to reduce morbidity and mortality in patients with heart failure and a reduced ejection fraction (HFrEF) (SOR A). ACE inhibitors are the preferred agent for inhibiting the renin-angiotensin system in patients with HFrEF, with ARBs generally reserved for those who are ACE inhibitor intolerant (SOR A). Although diuretics have been shown to improve symptoms of heart failure, a beneficial effect on morbidity and mortality has not been demonstrated. Combination therapy with an ACE inhibitor, ARB, and aldosterone antagonist is potentially harmful for patients and should not be routinely used (SOR C).In patients with chronic symptomatic HFrEF and an elevated BNP who tolerate an ACE inhibitor or ARB, use of an ARB combined with an angiotensin receptor-neprilysin inhibitor (ARNI) such as sacubitril or valsartan was shown in the PARADIGM-HF trial to further reduce morbidity and mortality, and is recommended in the American Heart Association (AHA) heart failure guidelines (SOR B). Although there is evidence that ivabradine can reduce hospitalizations for patients with symptomatic HFrEF on guideline-directed medical therapy, AHA guidelines recommend that it be considered only in patients with a heart rate >70 beats/min who are currently taking a maximally tolerated dosage of a β-blocker (SOR B).Both ACE and neprilysin break down bradykinin, a substance that directly or indirectly can cause angioedema. The combination of an ACE inhibitor and a neprilysin inhibitor has been associated with the development of angioedema. In light of this, the AHA recommends that an ARNI not be administered within 36 hours of switching from or to an ACE inhibitor (SOR C).

A 69-year-old female presents to the emergency department with a 1-hour episode of severe substernal chest pain that has now resolved. Her past medical history is notable for current tobacco abuse, hypertension, and depression. Her current medications include lisinopril/hydrochlorothiazide (Zestoretic), 10/12.5 mg daily; citalopram (Celexa), 20 mg daily; and aspirin, 81 mg daily. On examination she has a blood pressure of 150/92 mm Hg and a pulse rate of 92 beats/min. An EKG reveals a sinus rhythm with deep and symmetrical T-wave inversions in the inferior leads.You decide to admit the patient to the hospital. Which one of the following should be administered on admission? Alteplase (Activase) intravenously Aspirin, 81 mg, and nitroglycerin via intravenous drip Enoxaparin (Lovenox), 1 mg/kg subcutaneously, and nitroglycerin, 0.4 mg sublingually Ticagrelor (Brilinta), 60 mg orally, and enoxaparin, 1 mg/kg subcutaneously Ticagrelor, 180 mg, and aspirin, 325 mg

E The management of unstable angina or non-ST-elevation myocardial infarction (NSTEMI) is similar to the management of ST-elevation myocardial infarction except that fibrinolytic therapy has no role in unstable angina or NSTEMI (SOR A). Studies indicate that fibrinolytic therapy in these patients has no benefit in terms of mortality or myocardial infarction (MI), and may even increase the risk for intracranial hemorrhage and both fatal and nonfatal MI.Unless there is a contraindication, all patients with acute coronary syndrome should begin dual antiplatelet therapy with aspirin, starting with a loading dose of 325 mg followed by a maintenance dosage of 81 mg daily, and a P2Y12 inhibitor (either clopidogrel, prasugrel, or ticagrelor), as well as anticoagulation therapy with either low molecular weight heparin (SOR A), fondaparinux in combination with a factor IIa inhibitor (SOR B), unfractionated heparin (SOR B), or bivalirudin in patients managed with an early invasive strategy (SOR B). β-Blockers have been shown to reduce myocardial ischemia, reinfarction, and the frequency of complex ventricular dysrhythmias, and they increase long-term survival. Provided there are no contraindications, American Heart Association guidelines recommend that oral β-blocker therapy be initiated within the first 24 hours in patients with acute coronary syndrome (SOR A).

A 74-year-old female presents with a 2-month history of increased dyspnea on exertion. She was a long-time cigarette smoker but quit 20 years ago. She has COPD treated with inhaled ipratropium (Atrovent), a combination inhaled corticosteroid, and a long-acting β-agonist. She can walk up one flight of stairs in her home but for the past 2 months she has had to stop and rest before reaching the top due to increased dyspnea. She also has severe osteoarthritis of the left hip treated with acetaminophen, 1000 mg three times daily, and tramadol (Ultram), 50 mg twice daily. A resting EKG in the office is normal.Which one of the following would be the most appropriate initial study to evaluate this patient for ischemic heart disease? A treadmill exercise test An adenosine technetium 99m test A dipyridamole thallium test Treadmill echocardiography Dobutamine echocardiography

E This patient's severe degenerative joint disease would likely limit her ability to exercise sufficiently to achieve 85% of her expected heart rate, which is required for an adequate treadmill exercise test, unless the patient is on β-blocker therapy, which would allow 65% of the predicted heart rate to be considered adequate. Dipyridamole and adenosine are contraindicated for patients with severe asthma, COPD, hypotension, bradycardia, or heart block. A resting EKG and resting echocardiography might be appropriate, but would not rule out ischemic heart disease. Dobutamine provides a pharmacologic means to stress the heart in patients who cannot exercise. These agents enhance myocardial contractile performance and wall motion, thus making poorly functioning areas assessable by echocardiography. In patients unable to exercise to the target heart rate, pharmacologic agents are needed to complete stress testing.

A 58-year-old male is hospitalized with severe decompensated heart failure refractory to intravenous inotropic therapy and guideline-directed medical therapy. You are considering referral to a tertiary care hospital for mechanical circulatory support to bridge to transplantation.Which one of the following is true regarding mechanical circulatory support bridge therapy? It should be limited to patients who meet the criteria for heart transplantation It should only be used in patients with biventricular heart failure It generally improves quality of life while waiting for transplantation It greatly reduces quality of life while waiting for transplantation

c Mechanical circulatory support (MCS) with a ventricular assist device has continued to evolve and has emerged as a viable therapeutic option for patients with advanced stage D heart failure with reduced ejection fraction refractory to guideline-directed medical therapy and cardiac device intervention. A variety of ventricular assist devices are now available. These devices may be either intracorporeal or extracorporeal, and may be designed to assist the left ventricle, right ventricle, or both.Bridge therapy refers to the use of left ventricular assist devices to help a patient survive until a donor heart becomes available for transplantation. Several devices are available, some of which are implantable and allow patients to be discharged to their homes. These devices can increase patient activity levels and quality of life. Complications can occur, including stroke, infection, and death, but these devices can be lifesaving in patients with refractory heart failure.The data from the Interagency Registry for Mechanically Assisted Circulatory Support indicates that cardiogenic shock, advanced age, and severe right heart failure (manifested as ascites or increased bilirubin) are major risk factors for death after MCS. This led to a recommendation that referral for MCS be considered before severe right ventricular failure develops. Possible indications for a bridge-to-candidacy ventricular assist device include obesity, tobacco use, and severe pulmonary hypertension in patients who might otherwise be candidates for transplantation.


Conjuntos de estudio relacionados

Passpoint Quality Improvement Block 3 MS ML6

View Set

Anatomy and Physiology - Lesson 1

View Set

Combo with "Genetics Ch 5 Genetic Linkage and Mapping in Eukaryotes" and 14 others

View Set

Things Fall Apart characters from chapter 1-7

View Set

Genghis Khan and the Making of the Modern World

View Set

Customer Accounts: Opening Procedures For Other Account Types

View Set

Organization Strategy and Project Selection (Chapter 2)

View Set